Tải bản đầy đủ (.pdf) (46 trang)

Một số vấn đề về bài toán đếm trong tổ hợp (Luận văn thạc sĩ)

Bạn đang xem bản rút gọn của tài liệu. Xem và tải ngay bản đầy đủ của tài liệu tại đây (462.17 KB, 46 trang )

ĐẠI HỌC THÁI NGUYÊN
TRƯỜNG ĐẠI HỌC KHOA HỌC
---------------------------

NGUYỄN THỊ BÍCH PHƯỢNG

MỘT SỐ VẤN ĐỀ
VỀ BÀI TOÁN ĐẾM TRONG TỔ HỢP

LUẬN VĂN THẠC SĨ TOÁN HỌC

THÁI NGUYÊN - 2018


ĐẠI HỌC THÁI NGUYÊN
TRƯỜNG ĐẠI HỌC KHOA HỌC
---------------------------

NGUYỄN THỊ BÍCH PHƯỢNG

MỘT SỐ VẤN ĐỀ
VỀ BÀI TOÁN ĐẾM TRONG TỔ HỢP
Chuyên ngành: Phương pháp Toán sơ cấp
Mã số: 8460113

LUẬN VĂN THẠC SĨ TOÁN HỌC

NGƯỜI HƯỚNG DẪN KHOA HỌC
TS. Hoàng Lê Trường

THÁI NGUYÊN - 2018




i

Mục lục
MỞ ĐẦU
Chương 1. Bài toán đếm

ii
1

1.1

Định lí nhị thức . . . . . . . . . . . . . . . . . . . . . . . . . . . . .

1

1.2

Lựa chọn với sự lặp lại . . . . . . . . . . . . . . . . . . . . . . . . .

5

1.3

Phân hoạch . . . . . . . . . . . . . . . . . . . . . . . . . . . . . . . .

7

1.4


Đếm lặp . . . . . . . . . . . . . . . . . . . . . . . . . . . . . . . . .

7

1.5

Nguyên tắc trung bình . . . . . . . . . . . . . . . . . . . . . . . . . 12

1.6

Nguyên tắc bao hàm loại trừ . . . . . . . . . . . . . . . . . . . . . . 15

Chương 2. Đếm nâng cao

19

2.1

Chặn cỡ của các tập giao . . . . . . . . . . . . . . . . . . . . . . . . 19

2.2

Đồ thị không có chu trình độ dài 4 . . . . . . . . . . . . . . . . . . 23

2.3

Vấn đề của Zarankiewicz . . . . . . . . . . . . . . . . . . . . . . . . 32

2.4


Tính trù mật của ma trận nhị phân . . . . . . . . . . . . . . . . . 36

KẾT LUẬN

39

TÀI LIỆU THAM KHẢO

40


ii

MỞ ĐẦU
Toán học tổ hợp là một trong những nội dung quan trọng trong giáo dục phổ
thông. Học sinh thường gặp khó khăn khi giải quyết các bài toán này. Vì vậy,
tìm hiểu sâu thêm về toán tổ hợp là rất cần thiết. Trong toán học tổ hợp, bài
toán đếm là một trong những bài toán cơ bản. Phép đếm là một công cụ đắc
lực trong toán học và là một điều rất tự nhiên trong cuộc sống con người. Nhiều
kết quả đã biết trong toán học tổ hợp đều có thể được giải thích chỉ bằng phép
đếm.
Luận văn được viết dựa chủ yếu trên tài liệu chính để tham khảo là [4]. Mục
đích chính của luận văn là trình bày, khẳng định lại các kết quả đã có trong
toán học tổ hợp bằng lí luận đếm từ cơ bản đến nâng cao, phục vụ cho công
việc giảng dạy môn toán tổ hợp ở bậc THPT.
Cấu trúc luận văn gồm 2 chượng.
Chương 1. Bài toán đếm. Chương 1 trình bày lại các kiến thức cơ bản nhất
của bài toán đếm trong tổ hợp, cùng với một số ứng dụng điển hình của chúng
thông qua lí luận đếm. Đó là định lí khai triển nhị thức NewTon, lựa chọn với

sự lặp lại, phân hoạch, đếm lặp, nguyên tắc trung bình, nguyên tắc bao hàm
loại trừ.
Chương 2. Đếm nâng cao. Trên cơ sở vận dụng các kiến thức cơ bản đã được
trình bày trong chương 1, chương 2 trình bày một số kết quả nâng cao đã nghiên
cứu được từ bài toán đếm. Mục đích chính của chương 2 tập trung vào khai thác
một số kết quả quan trọng trong lí thuyết đồ thị. Đó là chặn cỡ của các tập
giao, đồ thị không có chu trình độ dài 4, vấn đề của Zarankiewicz, tính trù mật
của ma trận nhị phân.
Trong suốt quá trình làm luận văn, tác giả nhận được sự hướng dẫn và giúp


iii

đỡ tận tình của tiến sĩ Hoàng Lê Trường. Tác giả xin bày tỏ lòng biết ơn chân
thành và sâu sắc nhất tới thầy.
Tác giả cũng bày tỏ lòng biết ơn chân thành tới quí thầy cô giảng dạy lớp
cao học toán khóa 10, trường Đại Học Khoa Học - Đại Học Thái Nguyên đã
giảng dạy và truyền thụ đến cho tác giả nhiều kiến thức và kinh nghiệm nghiên
cứu khoa học.
Tác giả xin chân thành cảm ơn tới Ban giám hiệu, các đồng nghiệp trường
THPT Lí Nhân Tông, gia đình và bạn bè đã luôn động viên giúp đỡ và tạo điều
kiện cho tác giả về mọi mặt trong suốt quá trình học tập và thực hiện luận văn
này.
Thái Nguyên, ngày... tháng... năm 2018
Tác giả

Nguyễn Thị Bích Phượng


1


Chương 1. Bài toán đếm
1.1

Định lí nhị thức

Cho một tập có n phần tử, bài toán tìm số các tập con của nó có đúng k phần
tử là bài toán cổ nhưng quan trọng với học sinh THPT. Con số này (số các tập
con gồm k phần tử của tập hợp có n phần tử) thường được kí hiệu là Cnk và được
gọi là hệ số nhị thức. Nói cách khác, Cnk là số khả năng chọn ra k phần tử phân
biệt từ một bộ n phần tử phân biệt. Đẳng thức sau đây được chứng minh bởi
Sir Isaac Newton năm 1666, và được biết đến như định lí khai triển nhị thức
Newton.
Định lí khai triển nhị thức Newton. Cho n là một số nguyên dương. Khi
đó với mọi x, y , ta có

n
n

Cnk xk y n−k .

(x + y) =
k=0

Chứng minh. Ta có:
(x + y)n = (x + y)(x + y) · · · (x + y),
n số hạng

Để chứng minh định lí ta cần chứng minh với mọi k = 0, 1, ..., n ta thu được Cnk
số hạng xk y n−k .

Thật vậy, ta xét bài toán tìm số các số hạng có chứa xk trong khai triển. Một
cách tự nhiên chúng ta sẽ thực hiện phép đếm như sau. Đầu tiên, ta chọn k thừa
số (x + y) bất kì trong n thừa số (các thừa số giống nhau đều có dạng là (x + y))
ta có Cnk cách chọn. Tiếp theo, ứng với mỗi cách chọn đó, muốn có số hạng xk
ta lấy số x trong từng thừa số được chọn nhân với nhau thì ta sẽ được xk , số xk


2

này lại được tiếp tục nhân với nhóm còn lại, gồm (n − k) thừa số (x + y) hay nói
cách khác xk còn được nhân thêm với (x + y)n−k .
Vì số mũ của x chỉ được phép bằng k nên tiếp theo ta cần chọn xem trong khai
triển
(x + y)n−k = (x + y)(x + y) · · · (x + y),
(n−k) số hạng

Những số hạng nào nhân với xk mà sẽ không làm thay đổi số mũ của x. Ta thấy
ngay rằng, đó phải là các số hạng không chứa x, chỉ chứa y. Muốn tìm các số
hạng chỉ chứa y trong khai triển trên, thì chỉ có một cách duy nhất là ta lấy số
y trong (n − k) thừa số ấy nhân với nhau, và kết quả ta được số hạng y n−k .
Do đó, xk y n−k là số hạng mà chứa xk ứng với mỗi một cách chọn bộ k thừa số
(x + y).

Vì vậy, với Cnk cách chọn bộ k thừa số (x + y) như trên, ta thu được Cnk số
hạng có dạng xk y n−k .
Chú ý rằng định lí này chỉ là sự tổng quát hóa của hằng đẳng thức mà chúng
ta đã biết:
(x + y)2 = x2 + 2xy + y 2 .

Mặc dù đơn giản như vậy, nhưng định lí khai triển nhị thức có rất nhiều ứng

dụng.
Ví dụ 1.1. (Tính chẵn lẻ). Đây là một ví dụ điển hình, hãy chỉ ra tính chất sau
đây của các số nguyên: Nếu n,k là các số tự nhiên thì nk là số lẻ khi và chỉ khi
n là số lẻ.
Lời giải. (⇒) Nếu n = 2m, n là số chẵn thì nk = 2k mk cũng phải là số chẵn.
(⇐) Để chứng minh chiều ngược lại ta giả sử rằng n là số lẻ tức là n có dạng
n = 2m + 1 (m∈ N).

Áp dụng định lí khai triển nhị thức ta có:
nk = (2m + 1)k = 1 + (2m)1 Ck1 + (2m)2 Ck2 + · · · + (2m)k Ckk .

Từ khai triển trên ta thấy nk phải là số lẻ.


3

n giai thừa kí hiệu n! là tích của n số tự nhiên liên tiếp đầu tiên,
n! = n(n − 1) · · · 2.1

Điều này có thể mở rộng cho tất cả các số nguyên không âm với quy ước 0! = 1.
Chỉnh hợp chập k của của n là tích của k thừa số đầu tiên.
Akn =

n!
= n(n − 1) · · · (n − k + 1).
(n − k)!

Chú ý rằng Cn0 = 1 (tập rỗng) và Cnn = 1 (tập chứa toàn bộ n phần tử). Nói
chung, hệ số nhị thức có thể được viết dưới dạng thương của các giai thừa.
Mệnh đề 1.2. Cnk =


Akn
n!
=
.
k!
k!(n − k)!

Chứng minh. Ta thấy Akn là số các dãy có thứ tự (x1 , x2 , · · · , xk ) gồm k phần tử
khác nhau của một tập hợp cố định có n phần tử. Có n cách chọn phần tử đầu
tiên x1 . Có (n − 1) cách chọn phần tử tiếp theo x2 ,· · ·
Bằng một cách khác ta có thể chứng minh được. Ta chọn một tập hợp k phần
tử khác nhau từ tập hợp có n phần tử, ta có Cnk cách chọn. Ứng với mỗi cách
chọn trên ta có Akk = k! cách sắp xếp thứ tự các phần tử của dãy (x1 , x2 , · · · , xk ).
Từ đó ta kết luận: Akn = Cnk k!.
Có nhiều đẳng thức hữu ích về mối quan hệ giữa các hệ số nhị thức. Trong
nhiều tình huống, sử dụng những điều rất tự nhiên thuộc về tổ hợp để chứng
minh các đẳng thức đại số giống như mệnh đề trên, chúng ta có thể thu được
kết quả mong muốn một cách dễ dàng. Ví dụ khi chúng ta thấy rằng mỗi tập
con được xác định một cách duy nhất theo phần bù của nó thì ngay lập tức ta
thu được các đẳng thức.
Cnn−k = Cnk .

Với đẳng thức này, và với n cố định, giá trị của hệ số nhị thức Cnk tăng đến
giữa rồi lại giảm. Dùng định lí khai triển nhị thức thì tổng của tất cả (n + 1) hệ
số bằng 2n , 2n chính là số tất cả các tập con của tập có n phần tử:
n

n


Cnk
k=0

Cnk (1)k (1)n−k = (1 + 1)n = 2n .

=
k=0


4

Bằng một cách tương tự dùng tổ hợp ta có thể thiết lập được các đồng nhất
thức hữu ích.
Mệnh đề 1.3. (Tam giác Pascal). Với mọi số nguyên n≥ k ≥ 1, ta có
k−1
k
Cnk = Cn−1
+ Cn−1
.
k−1
Chứng minh. Số hạng đầu Cn−1
là số các tập hợp có k phần tử trong đó luôn
k
có một phần tử cố định. Số hạng thứ hai Cn−1
là số các tập hợp có k phần tử

khác phần tử cố định trên. Do đó, tổng của chúng chính là Cnk .
Thay đổi n,k, tính một cách chính xác hệ số nhị thức Cnk là rất phức tạp.
Tuy nhiên, trong những ứng dụng, chúng ta thường chỉ quan tâm đến tỉ lệ tăng
của chúng sao cho đủ để ước lượng. Ví dụ như ước lượng có thể thu được bằng

cách sử dụng khai triển Taylor của hàm số mũ và hàm số lôgarit.
(2)

et = 1 + t +

t2 t3
+ + ··· ,
2! 3!

∀t ∈ R


(3)

ln (1 + t) = t −

t2 t3 t4
+ − + ··· ,
2
3
4

∀ − 1 < t ≤ 1.

Điều này đặc biệt kéo theo một số ước lượng hữu ích khác
1 + t < et ,

(4)

1−t>e


(5)

Mệnh đề 1.4. (6) : ( nk )k ≤ Cnk và

k
i=0

∀t = 0,

2
−t− t2

,

∀0 < t < 1.

k
Cni ≤ ( en
k ) .

Chứng minh. Bị chặn dưới:
n
nn
n
nn−1
n−k+1
( )k =
··· ≤
···

= Cnk .
k
kk
k
kk−1
1

Bị chặn trên: Cho 0 < t ≤ 1, từ định lí nhị thức ta có bất đẳng thức sau:
k

k

Cni
i=0

Cni


i=0

ti
(1 + t)n
=
.
tk
tk


5
k

n

Tiếp theo ta thay t = .

Những ước lượng chặt chẽ hơn sau đây có thể thu được từ công thức nổi
tiếng Stirling cho giai thừa:
(7)

n √
n! = ( )n 2πneαn ,
e

ở đây 1/(12n + 1) < αn < 1/12n. Ước lượng này là cơ bản và có nhiều ứng dụng
ví dụ như để tính xấp xỉ cho chỉnh hợp chập k của n phần tử
Akn = nk e

(8)

−k2
k3
− 6n
2 +o(1)
2n

với k = o(n3/4 ),

và vì thế, áp dụng đối với hệ số tổ hợp ta có:
Cnk =

(9)


1.2

nk e

−k2
k3
− 6n
2
2n

k!

(1 + o(1)).

Lựa chọn với sự lặp lại

Trong phần trước chúng ta xét số cách chọn r phần tử phân biệt từ một tập
hợp có n phần tử. Một điều tự nhiên đặt ra là điều gì sẽ xảy ra nếu chúng ta
chọn những phần tử giống nhau lặp lại. Nói cách khác, chúng ta có thể trả lời
câu hỏi có bao nhiêu nghiệm nguyên không âm của phương trình: x1 +· · ·+xn = r
(xi ≥ 0, ∀i = 1, · · · , n) (xem xi như là số lần mà phần tử thứ i được chọn). Công

thức sau đây của vấn đề này được đưa ra bởi Lov a´sz , P elika´n và Vesztergombi.
Lov a´sz đã giải bài toán chia kẹo bằng phương pháp lựa chọn với sự lặp lại.
Giả sử chúng ta có r cái kẹo cùng loại (giống nhau) chúng ta muốn chia cho
n đứa trẻ. Chúng ta có bao nhiêu cách chia ?. Ta kí hiệu xi là số kẹo mà chúng
ta phát cho đứa trẻ thứ i. Câu hỏi này tương đương với phát biểu ở trên.
Câu trả lời phụ thuộc vào chúng ta có bao nhiêu cái kẹo và chúng ta phải
công bằng như thế nào. Nếu chúng ta công bằng nhưng chúng ta chỉ có r < n

cái kẹo thì một điều tự nhiên là không có sự lặp lại và ta chỉ phát được cho mỗi
đứa trẻ không nhiều hơn một cái kẹo (đứa trẻ xi sẽ nhận được 0 hoặc 1 cái kẹo).
Trong trường hợp này câu trả lời thật dễ dàng. Chúng ta chỉ cần chọn r đứa


6

trẻ trong n đứa trẻ để mỗi đứa trẻ này nhận được 1 chiếc kẹo. Chúng ta sẽ thấy
rằng có Cnr cách làm như vậy.
Bây giờ giả sử rằng, chúng ta có đủ kẹo, r ≥ n. Chúng ta phải công bằng, đó
là chúng ta muốn mỗi đứa trẻ đều nhận được ít nhất một cái kẹo. Chúng ta bố
trí những chiếc kẹo thành một hàng có chiều dài r (từ trái qua phải) và để cho
đứa trẻ đầu tiên lấy kẹo từ trái sang phải. Sau một lúc, chúng ta dừng lại và để
cho đứa trẻ thứ hai chọn kẹo cứ làm như vậy mãi. Việc chia kẹo được xác định
bằng cách ghi rõ địa điểm, vị trí (giữa các kẹo liên tiếp nhau) của chỗ mà bắt
đầu với một đứa trẻ mới. Có r − 1 vị trí như vậy và chúng ta phải chon n − 1 vị
trí của chúng (đứa trẻ đầu tiên thường bắt đầu ở vị trí đầu tiên, vì vậy chúng
ta không chọn vị trí này). Ví dụ nếu chúng ta có n = 6 đứa trẻ và có r = 9 cái
kẹo, thì một tình huống điển hình như thế này:
λ2

λ3 λ4

λ5 λ6

Do đó, chúng ta phải chọn một tập (n − 1) phần tử từ một tập có (r − 1)
n−1
. Nếu chúng ta không công bằng thì chúng ta sẽ
phần tử. Số cách chọn là Cr−1


có nhiều cách chia hơn.
Mệnh đề 1.5. Số nghiệm nguyên của phương trình x1 + x2 + ... + xn = r với điều
r
kiện xi ≥ 0, với mọi i = 1, ..., n là Cn+r−1
.

Chứng minh. Trong tình huống này, chúng ta không công bằng và cho phép rằng
một vài đứa trẻ có thể không nhận được chiếc kẹo nào. Với những cách sau đây,
chúng ta có thể làm giảm được vấn đề đếm số các cách chia như vậy, chúng ta
chỉ cần làm như sau. Chúng ta mượn mỗi đứa trẻ một chiếc kẹo và sau đó chúng
ta chia toàn bộ số kẹo (n + r) chiếc cho những đứa trẻ sao cho mỗi đứa trẻ nhận
được ít nhất một cái kẹo. Bằng cách này, mọi đứa trẻ đều nhận lại được chiếc
kẹo mà chúng ta đã mượn chúng và một số đứa trẻ may mắn hơn sẽ được nhận
nhiều kẹo hơn. Phần hơn này chính là do từ r chiếc kẹo được chia cho n đứa
trẻ. Chúng ta đã biết số cách chia (n + r) chiếc kẹo cho n đứa trẻ một cách công
n−1
r
bằng như trên là Cn+r−1
hay chính là Cn+r−1
.


7

1.3

Phân hoạch

Một sự phân hoạch của n phần tử là một bộ những tập con rời nhau, gọi là
các khối sao cho hợp của chúng chính là toàn bộ tập có n phần tử ban đầu. Ta

kí hiệu S(n; k1 , k2 , · · · , kn ) là số tất cả các phân hoạch của n phần tử thành ki
khối,
(i = 1, 2, · · · , n; k1 + 2k2 + · · · + nkn = n).
ki = số các khối sao cho mỗi khối có i phần tử trong một phân hoạch.

Mệnh đề 1.6.
S(n; k1 , k2 , ..., kn ) =

n!
.
k1 !...kn !(1!)k1 ...(n!)kn

Chứng minh. Chúng ta thấy rằng bất kì một sự sắp xếp nào tức là một hoán
vị của n phần tử thì chúng ta sẽ thu được một phân hoạch như vậy. Bằng cách
làm như sau: Đầu tiên ta coi các k1 khối 1 phần tử như là các hộp được đánh
số từ 1 đến k1 , tiếp theo ta coi các k2 khối 2 phần tử như là các hộp được đánh
số từ k1 + 1 đến k2 ,. . . . Chúng ta có n! cách sắp xếp. Ta sẽ chỉ ra rằng với mỗi
phân hoạch như vậy sẽ được đếm k1 ! · · · kn !(1!)k1 · · · (n!)kn lần.
Thật vậy, chúng ta có thể xây dựng một sự sắp xếp của các phần tử như sau:
Đầu tiên ta xếp các khối có 1 phần tử, sau đó xếp các khối có 2 phần tử,...Chúng
ta có ki ! cách để sắp xếp các khối có i phần tử và có (i!)ki cách để xếp các phần
tử trong khối có i phần tử.

1.4

Đếm lặp

Nguyên tắc cơ bản của phép đếm lặp bắt nguồn từ các quan sát thực tế cơ
bản sau: Nếu các phần tử của một tập hơp được đếm bằng hai cách
khác nhau, thì kết quả thu được là giống nhau.

Trong hệ thống các ma trận 0 − 1 đều theo nguyên tắc cơ bản sau. M là một
ma trận 0 − 1 cỡ n × m, ri là số các số 1 trong dòng i, cj là số các số 1 trong cột
n

j. Thế thì

m

cj = tổng số các số 1 trong ma trận M .

ri =
i=1

j=1

Ví dụ sau đây là một minh họa cho nguyên tắc cơ bản của phép đếm lặp.


8

Giả sử rằng có một số lượng hữu hạn người gặp nhau trong một bữa tiệc và họ
bắt tay nhau. Giả sử rằng không có một ai tự bắt tay mình và hơn nữa là không
có hai người nào bắt tay nhau hơn một lần.
Bổ đề bắt tay: Tại một bữa tiệc, số khách mà mỗi người khách có số lần bắt
tay là số lẻ, là số chẵn.
Chứng minh. Gọi P1 , · · · , Pn là những người đến dự tiệc. Chúng ta ứng dụng
phép đếm lặp cho tập các cặp có thứ tự (Pi , Pj ) mà Pi và Pj bắt tay nhau ở bữa
tiệc. Gọi xi là số lần mà Pi bắt tay và y là số cái bắt tay đã xảy ra. Một mặt, số
n


các cặp bắt tay nhau là

xi , và với mỗi Pi số lựa chọn của Pj cũng là xi . Mặt
i=1

khác, mỗi một cái bắt tay được tính cho cả hai cặp (Pi , Pj ) và (Pj , Pi ), vì vậy
n

xi = 2y . Không mất tính tổng quát, giả sử

tổng số cái bắt tay là 2y. Do đó,
i=1

xi là số lẻ với mọi i = 1, . . . , n mà 2y lại là số chẵn suy ra n phải là số chẵn.

Bổ đề này cũng chính là một hệ quả trực tiếp của trường hợp tổng quát sau
đây mà nó chính là một phiên bản đặc biệt cho đồ thị, và đã được chứng minh
bởi Euler. Cho một điểm x, bậc của nó là số d(x) trong một họ F , nó chính là
số các phần tử của F chứa x.
Mệnh đề 1.7. Cho F là một họ các tập con của X. Khi đó
(10)

|A|.

d(x) =
x∈X

A∈F

Chứng minh. Xét ma trận liên thuộc M =(mx,A ) của F . M là ma trận 0 − 1 cùng

với |X| dòng, mỗi dòng được gán nhãn bởi x ∈ X và |F| cột được gán nhãn bởi
các tập A ∈ F sao cho (mx,A ) = 1 nếu và chỉ nếu x ∈ A. Quan sát ta thấy d(x)
chính là số của số 1 trong dòng x và |A| chính là số của số 1 trong cột A.
Đồ thị là một cấu trúc rời rạc gồm các đỉnh và các cạnh nối các đỉnh đó, nó
được mô tả hình thức:
G = (V, E)

V gọi là tập các đỉnh và E gọi là tập các cạnh. Có thể coi E là tập các cặp (u,v)
với 2 đỉnh u,v thuộc V. Và tùy theo đặc tính và số lượng của tập các cạnh E mà
người ta có thể phân loại đồ thị thành đơn đồ thị, đa đồ thị, đồ thị có hướng và


9

đồ thị vô hướng.
Như vậy, các đồ thị là họ các tập hợp gồm 2 phần tử, và bậc của đỉnh x là số
các cạnh liên thuộc với x, là số đỉnh trong lân cận của nó. Do đó, kéo theo ta
có định lí sau.
Định lý 1.8. (Euler1736). Trong mọi đồ thị, tổng số bậc của các đỉnh bằng 2
lần tổng số cạnh của nó và do đó nó là số chẵn.
Những đồng nhất thức sau đây có thể được chứng minh bằng cách tương tự
(11)

|Y ∩ A|,

d(x) =
x∈Y

A∈F
2


(12)

∀Y ⊂ X.

d(x) =
x∈X

|A ∩ B|.

d(x) =
A∈F x∈A

A∈F B∈F

Chứng minh (11). Tương tự, xét ma trận liên thuộc M =(mx,A ) của F . M là ma
trận 0 − 1 cùng với |Y | dòng, mỗi dòng được gán nhãn bởi x ∈ Y và |F| cột được
gán nhãn bởi các tập A ∈ F sao cho (mx,A ) = 1 nếu và chỉ nếu x ∈ A. Quan sát
ta thấy, d(x) chính là số của số 1 trong dòng x và |A ∩ Y | chính là số của số 1
trong cột A.
d(x)2 .

|A ∩ B| =

Chứng minh (12). Trước tiên ta chứng minh:
A∈F B∈F

x∈X

Thật vậy, ta có thể viết vế trái của công thức dưới dạng khác

|A ∩ B| =

S=
A∈F B∈F

1
x∈X x∈Ai ∩Aj
Ai ,Aj ∈F

Như vậy, để tính tổng S ở vế trái chúng ta đi đếm xem với mỗi x ∈ X có bao
nhiêu cặp (Ai , Aj ) của họ F mà x ∈ Ai ∩ Aj .
Ta cần chú ý bộ chỉ số (i, j) ở đây không nhất thiết phân biệt, và có thứ tự.
Đặt J = {i : x ∈ Ai , Ai ∈ F}.
Ta thấy, theo định nghĩa về bậc của x trong họ F thì d(x) = |J|.
Bây giờ, ta sẽ xem với mỗi x ∈ X nếu ta thực hiện phép đếm như trên, thì sự
đóng góp của nó vào tổng S là bao nhiêu.
Nếu J = ∅ tức là d(x) = 0 thì sẽ không có cặp tập hợp nào của F , mà có giao
của chúng có chứa x. Do đó, khi thực hiện việc đếm như trên, phần tử x sẽ cho


10

đóng góp vào tổng S ở trên là 0 = d(x)2 .
Nếu J = ∅ tức là d(x) = 0. Khi đó, số các cặp tập hợp (Ai , Aj ) của họ F với
i, j không nhất thiết phân biệt và có thứ tự, sao cho x ∈ Ai ∩ Aj chính là d(x)2 .
Điều này có nghĩa là khi thực hiện việc đếm như trên, phần tử x trong trường
hợp này cho đóng góp vào tổng S ở trên là d(x)2 .
Vì vậy,
1 = d(x)2
x∈Ai ∩Aj

Ai ,Aj ∈F

Do đó, lấy tổng trên tất cả các x ∈ X ta thu được
|A ∩ B| =

S=

x∈X x∈Ai ∩Aj
Ai ,Aj ∈F

A∈F B∈F

d(x)2 .

1=
x∈X

Tiếp theo, ta chứng minh công thức
|A ∩ B| =
A∈F B∈F

d(x).
A∈F x∈A

Thật vậy, theo công thức (11) với mỗi A cố định A ∈ F , ta có
|A ∩ B| =

d(x)

B∈F


x∈A

Vì vậy,
|A ∩ B| =
A∈F B∈F

d(x)
A∈F x∈A

Số Turan T(n,k,l) là số các tập con nhỏ nhất có l phần tử của tập X có n
phần tử mà mọi tập con của X có k phần tử đều chứa ít nhất một tập trong
chúng.
Mệnh đề 1.9. Với mọi số nguyên dương l ≤ k ≤ n,
T (n, k, l) ≥

Cnl
.
Ckl

Chứng minh. Gọi F là một họ nhỏ nhất gồm các tập hợp có l phần tử trên X
mà mọi tập con của X có k phần tử đều chứa ít nhất một phần tử của F . Đặt


11

ma trận 0 − 1 là ma trận M =(mA,B ) mà các dòng của nó được gán nhãn bởi các
tập A trong F , các cột được gán nhãn bởi các tập con B của X có k phần tử,
và mA,B = 1 nếu và chỉ nếu A ⊆ B.
Gọi rA là số của số 1 trong dòng A và cB là số của số 1 trong cột B . Thế thì

cB ≥ 1 với mọi B vì B phải chứa ít nhất một tập hợp trong F . Mặt khác, rA là

số các tập con B có k phần tử trong đó B luôn chứa một tập có l phần tử cố
k−l
định. Vì vậy, rA = Cn−l
với mọi A ∈ F . Bằng nguyên tắc đếm lặp,

k−l
|F|Cn−l
=

cB ≥ Cnk ,

rA =
A∈F

B


T (n, k, l) = |F| ≥

Cnk
k−l
Cn−l

=

Cnl
,
Ckl


Bất đẳng thức cuối này chính là một tính chất khác của hệ số nhị thức.

Các ứng dụng của phép đếm lặp sau đây là tới từ lý thuyết số: Có bao nhiêu
số là ước của số tự nhiên n. Nếu t(n) là số các ước của n thì dáng điệu của hàm
này không ổn định: t(p) = 2 với mọi số nguyên tố p, trong khi đó t(2m ) = m + 1.
Do đó, một điều thú vị là ta xét số trung bình của số các ước số:
τ (n) =

t(1) + t(2) + · · · + t(n)
thì con số này hoàn toàn ổn định. Nó xấp xỉ ln n.
n

Mệnh đề 1.10. |τ (n) − ln n| ≤ 1.
Chứng minh. Ứng dụng nguyên tắc đếm cơ bản, xét ma trận 0-1 cỡ n × n,
M = (mij ) với mi,j = 1 khi và chỉ khi j chia hết cho i: Số lần xuất hiện của

số 1 trong cột thứ j chính là t(j), là số các ước số của j . Vì vậy, lấy tổng
trên các cột chúng ta thấy tổng của số lần xuất hiện số 1 trong ma trận là
T (n) = t(1) + t(2) + · · · + t(n).

Mặt khác, số các số 1 trong dòng i là số bội số của i: i, 2i, 3i, ..., ri mà ri ≤ n.
n
i

Do đó, chúng ta có [ ] chữ số 1 trong dòng i. Lấy tổng trên các dòng ta thu
được

n


Tn =
i=1

n
[ ]. Vì x − 1 < [x] ≤ x, ∀x ∈ R nên chúng ta thu được :
i


12

1
2
3
4
5
6

1

2

3

4

5

6

7


8

9

10

1

1

1

1

1

1

1

1

1

1

1

1


1

1

1

1

1
1

1

1
1

1
1

7

1

8

1

1
Tn ≤ Hn ,

n
1 1
1
Hn = 1 + + + · · · + = ln n + γn ,
2 3
n
Hn − 1 ≤ τ (n) =

ở đây (13)

0 ≤ γn ≤ 1

là số điều hòa thứ n.

1.5

Nguyên tắc trung bình

Giả sử rằng chúng ta có một tập hợp gồm m đối tượng (phần tử). Đối tượng
thứ i có cỡ là li , và chúng ta muốn chứng minh có ít nhất một đối tượng trong
tập hợp có cỡ lớn hơn t, với t cho trước nào đó, tức là tồn tại đối tượng i có lực
lượng li ≥ t với t cho trước. Trong tình huống này, chúng ta xét lực lượng trung
bình l =

li
m

và cố gắng chứng minh l ≥ t. Điều này ngay lập tức mang lại kết

quả sau.

Nguyên tắc trung bình. Mọi tập của các số phải chứa ít nhất một số lớn
hơn hoặc bằng số trung bình và ít nhất một số nhỏ hơn hoặc bằng số trung bình.
Nguyên tắc này rất đơn giản nhưng ứng dụng của nó thật đáng ngạc nhiên.
Chúng ta sẽ thường xuyên sử dụng nguyên tắc này. Sử dụng nguyên tắc này,
chúng ta cùng chứng minh điều kiện đủ sau đây về một đồ thị không liên thông.
Nhắc lại rằng một thành phần liên thông trong một đồ thị là một tập hợp các
đỉnh sao cho luôn có một đường đi giữa 2 đỉnh bất kì của chúng. Một đồ thị là
liên thông nếu nó chỉ chứa một thành phần liên thông, nếu không thì nó là đồ


13

thị không liên thông.
Mệnh đề 1.11. Mọi đồ thị với n đỉnh cùng với số cạnh nhỏ hơn n − 1 thì không
liên thông.
Chứng minh. Chúng ta chứng minh bằng quy nạp theo n. Khi n = 1, mệnh đề
là hiển nhiên vì không có đồ thị nào có số cạnh âm. Khi n = 2, một đồ thị cùng
với ít hơn 1 cạnh hiển nhiên là không liên thông.
Giả sử rằng mệnh đề đúng với đồ thị có n đỉnh. Cho đồ thị G = (V, E) có
|V | = n + 1 đỉnh sao cho |E| ≤ n − 1. Theo định lý Euler (Định lí 1.8) bậc trung
2|E|
2(n − 1)
1
d(x) =

< 2.
bình của tất cả các đỉnh là
|V | x∈V
|V |
n+1

Theo nguyên tắc trung bình, tồn tại một số đỉnh sẽ có bậc 0 hoặc 1. Nếu
d(x) = 0 thì x chính là một thành phần liên thông và phần còn lại có một thành

phần liên thông khác của G, vì vậy G không liên thông. Nếu d(x) = 1, giả sử rằng
đỉnh duy nhất kề với x là y . Thế thì đồ thị H thu được từ G bằng cách xóa đỉnh
x và các cạnh xy sẽ là đồ thị có |V | − 1 = n đỉnh và |E| − 1 ≤ (n − 1) − 1 = n − 2

cạnh. Theo giả thiết quy nạp, H không liên thông. Vì y thuộc vào duy nhất một
thành phần liên thông của H , gọi là P và thành phần liên thông khác không
chứa y là P trong H nên x cũng thuộc cùng thành phần P và không thuộc thành
phần liên thông P trong G. Do đó G cũng không liên thông.
Chúng ta đề cập đến một bất đẳng thức quan trọng mà chúng đặc biệt hữu
ích khi làm việc với những đại lượng trung bình.
Một hàm số f(x) nhận giá trị thực là hàm lồi nếu
f (λa + (1 − λ)b) ≤ λf (a) + (1 − λ)f (b), ∀0 ≤ λ ≤ 1.

Theo quan điểm hình học, tính lồi của hàm f có nghĩa là nếu chúng ta vẽ
một đường l nối điểm (a, f (a)) và điểm (b, f (b)) thì đồ thị f(z) phải nằm phía
dưới của l(z) với z ∈ [a; b]. Do đó, để hàm f là hàm lồi thì điều kiện đủ là đạo
hàm cấp 2 của hàm f phải không âm.
n

Mệnh đề 1.12. (Bất đẳng thức Jensen). Nếu 0 ≤ λi ≤ 1,

λi = 1 và f là hàm
i=1

lồi, thì
(14)


n

n

λi xi ) ≤

f(
i=1

λi f (xi ).
i=1


14

a

λa + (1 − λ)b

b

Hình 1.1: Một hàm lồi

Chứng minh. Ta dễ dàng chứng minh bằng quy nạp trên n.
Khi n = 2, mệnh đề đúng.
Giả sử bất đẳng thức đúng với số các số hạng tăng lên đến n, ta sẽ chứng minh
nó đúng với n + 1. Để chứng minh điều này ta chỉ việc thay thế 2 số hạng đầu
tiên trong biểu thức λ1 x1 + λ2 x2 + · · · + λn+1 xn+1 bằng biểu thức
λ1
λ2

x1 +
x2 ), và sử dụng giả thiết quy nạp.
λ1 + λ2
λ1 + λ2
1
Nếu a1 , · · · , an là các số không âm xét f (x) = x2 , và λi = ta thu được một bất
n
đẳng thức hữu ích (nó cũng là một hệ quả của bất đẳng thức Cauchy − Schwarz )

(λ1 + λ2 )(

n

a2i

(15)
i=1

1
≥ (
n

n

ai )2 .
i=1

Bất đẳng thức Jensen(15) mang lại bất đẳng thức hữu ích sau giữa số học và
hình học, cụ thể: Với bất kì các số không âm a1 , · · · , an , ta có:
1

n

(16)

n

n
1

ai ≥ (
i=1

ai ) n .
i=1

Để chứng minh điều này, sử dụng bất đẳng thức Jensen với f (x) = 2x ,
λ1 = · · · = λn =
1
n

1
và xi = log2 ai , ∀1 = 1, · · · , n. Thế thì:
n

n

n

λi f (xi ) ≥ f (


ai =
i=1

n

i=1

n

(

λi xi ) = 2
i=1

xi )/n
i=1

n

ai )1/n .

=(
i=1


15

1.6

Nguyên tắc bao hàm loại trừ


Nguyên tắc bao hàm loại trừ (Sàng Eratosthenes) là một công cụ đắc lực
trong lý thuyết liệt kê, lý thuyết số. Nguyên tắc này liên quan đến lực lượng của
hợp các tập hợp, lực lượng của giao các tập hợp, những lực lượng này chúng ta
có thể tính được một cách dễ dàng.
Cho A và B là 2 tập hợp bất kì.Ta có:
|A ∪ B| = |A| + |B| − |A ∩ B|,

Nói chung, khi chọn n tập con A1 , · · · , An của tập X, chúng ta muốn tính số
phần tử của |A1 ∪ · · · ∪ An |. Để tính xấp xỉ đầu tiên của số này, ta có thể đi tính
tổng
|A1 | + · · · + |An |.

(17)

Tuy nhiên, nói chung là con số này lớn hơn số phần tử của |A1 ∪ · · · ∪ An |.
Nếu Ai ∩ Aj = ∅ thì mỗi phần tử của Ai ∩ Aj được đếm 2 lần trong (17): một lần
trong |Ai | và một lần trong |Aj |. Chúng ta có thể xử lý điều này bằng cách trừ
đi từ (17) tổng
|Ai ∩ Aj |.

(18)
1≤i
Nhưng khi đó chúng ta lại thu được một con số nhỏ hơn với số phần tử của
|A1 ∪· · ·∪An | vì mỗi một phần tử trong Ai ∩Aj ∩Ak = ∅ được đếm 3 lần trong(18):

một lần trong |Ai ∩ Aj |, một lần trong | Aj ∩ Ak | và một lần trong |Ai ∩ Ak |.
Chúng ta có thể xử lý tình huống này bằng cách cộng thêm tổng
|Ai ∩ Aj ∩ Ak |,

1≤i
Nhưng chúng ta lại thu được một kết quả lớn hơn nhiều so với kết quả cần
tính...Cứ như vậy, sau n bước chúng ta sẽ nhận được kết quả chính xác. Kết quả
này được biết như nguyên tắc bao hàm, loại trừ.
Các ký hiệu sau đây sẽ trở nên tiện dụng: nếu I là tập con của tập chỉ số
{1, · · · , n}, ta đặt:
AI =

Ai ,
i∈I


16

với quy ước A∅ = X.
Mệnh đề 1.13. (Nguyên tắc bao hàm-loại trừ). Cho A1 , ..., An là các tập con
của X. Số các phần tử của X mà không nằm trong bất cứ tập con Ai nào là
(−1)|I| |AI |.

(19)

I⊆{1,...,n}

Chứng minh. Tổng trên là một tổ hợp tuyến tính của các lực lượng của các tập
AI với các hệ số là 1 và -1. Chúng ta có thể viết lại tổng đó như sau
(−1)|I| | AI |=
I

(−1)|I| =

I

(−1)|I| .

x∈AI

I

I:x∈AI

Chúng ta sẽ tính mỗi phần tử của X sẽ đóng góp vào tổng, tổng đó chính là
tổng các hệ số của các tập AI mà có chứa phần tử đó
Trước hết giả sử rằng x ∈ X không nằm trong bất cứ một tập Ai nào. Thế thì
số hạng trong tổng mà x đóng góp vào ứng với I = ∅ và đóng góp này bằng 1.
Ngược lại, tập J={i : x ∈ Ai } = ∅ và x ∈ AI chỉ khi I ⊆ J . Do đó, đóng góp của
x là
|J|

(−1)
I⊆J

|I|

i
(C|J|
)(−1)i = (1 − 1)|J| = 0

=
i=0


Do đó điểm mà không nằm trong bất cứ một tập Ai thì nó góp vào tổng là
1, trong khi đó những điểm nằm trong Ai thì nó đóng góp là 0. Vì vậy tổng
trên là số các phần tử mà không nằm trong bất cứ tập Ai nào, điều phải chứng
minh.
Mệnh đề 1.14. Cho A1 , ..., An là các tập hợp (không nhất thiết phân biệt). Khi
đó
(20)

|A1 ∪ ... ∪ An | =

(−1)|I|+1 |AI |.
∅=I⊆{1,...,n}

Chứng minh. Vế trái của (20) là |A∅ | trừ đi số phần tử của X = A∅ mà không
nằm trong bất kỳ một tập con Ai nào. Theo mệnh đề (1.13)ta có số này là:


17

(−1)|I| |AI | =

|A∅ | −

(−1)|I|+1 | AI |.
∅=I⊆{1,··· ,n}

I⊆{1,··· ,n}

Giả sử chúng ta muốn biết khi cho một tập chỉ số I, có bao nhiêu phần tử
mà thuộc về các tập Ai , ∀i ∈ I và không thuộc về bất cứ tập nào còn lại. Mệnh

đề 1.14 (tương ứng với trường hợp I = ∅) có thể được tổng quát hóa lên cho tình
huống này.
Mệnh đề 1.15. A1 , ..., An là các tập hơp và I là một tập con của tập chỉ số
{1, ..., n}. Khi đó số các phần tử thuộc Ai , ∀i ∈ I và không thuộc bất cứ một tập

nào khác là
(−1)|J\I| |AJ |.

(21)
J⊇I

Chứng minh. Xét tập X=

Ai , tập con của X là Bk = X ∩ Ak , ∀k ∈ N \ I , trong

i∈I

đó N = {1, · · · , n}.
Mệnh đề yêu cầu chúng ta tính số phần tử của X không nằm trong bất cứ tập
Bk nào. Theo mệnh đề (1.13) số này là:
(−1)|K| |
K⊆N \I

(−1)|K| |

Bk |=
k∈K

K⊆N \I


(−1)|J\I| |AJ |.

Ai | =
i∈K∪I

J⊇I

Xác suất được tính thế nào nếu n người ngẫu nhiên tìm trong một cái tủ tối
để lấy những chiếc mũ của họ, sao cho không có người nào lấy đúng mũ của
mình. Sử dụng nguyên tắc bao hàm và loại trừ, có thể thấy rằng xác suất này
rất gần e−1 = 0, 3678 · · ·
Câu hỏi này có thể được chính xác hóa như sau: Một hoán vị là một song
ánh f từ tập {1, · · · , n} vào chính nó. Chúng ta nói rằng f cố định phần tử i (i
là phần tử bất động) nếu f (i) = i. Một sự xáo trộn là một hoán vị mà không cố
định bất cứ phần tử nào. Chúng ta có n! hoán vị. Có bao nhiêu số xáo trộn?


18

Mệnh đề 1.16. Số xáo trộn của tập {1, ..., n} là
n

n

(−1)

(22)

i


Cni (n − i)!

= n!
i=0

i=0
n

Chứng minh. Tổng
i=0

(−1)i
.
i!

(−1)i
là phần đầu tiên của khai triển Taylor của e−1 . Vì
i!

vậy số xáo trộn xấp xỉ khoảng e−1 phần của số hoán vị.
Chúng ta sẽ ứng dụng công thức bao hàm loại trừ mệnh đề 1.13. Gọi X là tập
tất cả các hoán vị mà cố định phần tử i, vì vậy | Ai |= (n − 1)! và hơn nữa
| AI |= (n− | I |)! vì các hoán vị trong AI cố định mọi phần tử trong I và các

phần tử còn lại chọn tùy ý. Một hoán vị là một xáo trộn nếu và chỉ nếu nó không
nằm trong bất cứ một tập Ai nào, vì vậy theo công thức 19 số xáo trộn là:
n
|I|

(−1) |AI | =


|I|

i=0

I⊆{1,··· ,n}

I⊆{1,··· ,n}

(−1)i Cni (n − i)!

(−1) (n− | I |)! =

Thật vậy, đặt i = |I| thì rõ ràng i ∈ {0, 1, 2, · · · , n}. Ta có
AI =

Ai
i∈I

Ở đây, AI chính là số các hoán vị mà có |I| = i phần tử cố định. Ta sẽ đi đếm
số phần tử của AI như sau. Đầu tiên ta chọn i = |I| phần tử từ n phần tử
để làm phần tử cố định trong hoán vị, ta có Cni cách chọn. Ứng với mỗi cách
chọn này, ta lại có (n − i)! hoán vị mà đảm bảo có i phần tử là cố định. Do đó,
|AI | = Cni (n − i)!.

Theo công thức bao hàm - loại trừ ta có số xáo trộn bằng
(−1)|I| |AI | =
I⊆{1,···n}

=


(−1)|I| (n − |I|)!
I⊆{1,··· ,n}
n
i

(−1) Cni (n − i)!

i=0
n

= n!
i=0

(−1)i
.
i!


19

Chương 2.Đếm nâng cao
2.1

Chặn cỡ của các tập giao

Có bao nhiêu tập con có r phần tử của một tập có n phần tử với điều kiện
là không có 2 tập con nào trong chúng có giao nhiều hơn k phần tử?. Câu trả
lời cho vấn đề này được đưa ra bởi Corra´di năm 1969.
Mệnh đề 2.1. (Corra´di 1969). Cho A1 , ..., AN là các tập hợp có r phần tử và X

là hợp của chúng. Nếu | Ai ∩ Aj |≤ k, ∀i = j thì
| X |≥

(25)

r2 N
.
r + (N − 1)k

Chứng minh. Chỉ bằng công thức (11) ta có với mỗi i = 1, · · · , N
n

(26)

|Ai ∩ Aj | = |Ai | +

d(x) =
j=1

x∈Ai

|Ai ∩ Aj | ≤ r + (N − 1)k.
j=i

ở đây d(x) chính là bậc của x trong họ F = {A1 , · · · AN }.
Lấy tổng trên tất cả các tập Ai và sử dụng bất đẳng thức Jensen(1.12) ta thu
được
N

d(x) =

i=1 x∈Ai

1
d(x) ≥ (
n
2

x∈X

x∈X

1
d(x)) = (
n

N

(N r)2
| Ai |) =
.
n
2

2

i=1

Sử dụng (26) ta thu được : (N r)2 ≤ N |X|(r + (N − 1)k), chúng ta sẽ thu được
chặn dưới của |X| như mong muốn.
Cho một họ các tập A1 , · · · , AN , có lực lượng trung bình của chúng là


1
N

N

|Ai |.
i=1

Mệnh đề sau đây nói rằng nếu lực lượng trung bình của các tập lớn thì sẽ tồn
tại 2 tập trong chúng có chứa nhiều phần tử chung.


20

Mệnh đề 2.2. Cho X là một tập có n phần tử, và A1 , ..., AN là những tập con
của X mà có lực lượng trung bình ít nhất

n
. Nếu N ≥ 2ω 2 thì tồn tại i = j sao
ω

cho
| Ai ∩ Aj |≥

(27)

n
2ω 2


Chứng minh. Lại một lần nữa chúng ta sử dụng phương pháp đếm. Một mặt sử
dụng bất đẳng thức Jensen và bất đẳng thức (15) ta thu được:
1
d(x) ≥ (
n
2

x∈X

1
d(x)) = (
n

N

| Ai |)2 ≥

2

x∈X

i=1

nN 2
.
ω2

Mặt khác, giả sử rằng (27)là sai và sử dụng (12) cùng với bất đẳng thức |Ai ∩Aj | <
n
ta thu được

2ω 2
N

N

2

| Ai ∩ Aj |=

d(x) =
i=1 j=1

x∈X

< nN +

| Ai | +
i

| Ai ∩ Aj |
i=j

nN 2
2ω 2
1
nN 2
nN (N − 1)
=
(1
+


)

, mâu thuẫn.
2ω 2
2ω 2
N
N
ω2

Mệnh đề 2.2 là trường hợp đặc biệt của kết quả tổng quát sau:
Mệnh đề 2.3. (Erdo¨s 1964b). Cho X là tập có n phần tử x1 , ..., xn và A1 , ..., AN
là N tập con của X có lực lượng trung bình ít nhất
Ai1 , ..., Aik sao cho | Ai1 ∩ ... ∩ Aik |≥

n
.
2ω k

n
. Nếu N ≥ 2kω k thì tồn tại
ω

Chứng minh. Chứng minh mệnh đề 2.3 là sự tổng quát hóa của một trường hợp
ở trên.
Để chứng minh mệnh đề này chúng ta đi chứng minh bài toán tổng
quát sau đây liên quan đến bậc của một phần tử trong một họ F .
Bài toán. Cho F = {A1 , ..., AN } là một họ gồm các tập con của X. Chứng
minh rằng với mọi 1 ≤ s ≤ N ta có
d(x)s =

x∈X

|Ai1 ∩ Ai2 ∩ · · · ∩ Ais |.
(i1 ,··· ,is )

ở đây bộ chỉ số (i1 , · · · , is ) không nhất thiết phân biệt.


×